Hey , can someone please help me with this problem.

Hey , Can Someone Please Help Me With This Problem.

Answers

Answer 1

Answer:

Ok so for intrest you use the formula

I=prt

so we just multiply everthing together

800*0.03*6=144

She'll make 144 dollars

Hope This Helps!!!


Related Questions

the common multiple of 4 and 20 is? a.3 b.4 c.8 d.20

Answers

Answer:

i belive its A

Step-by-step explanation: hope this helps :)

Answer:

B. 4

Step-by-step explanation:

4 times 5 is 20 and 20 divided by 5 is 4

pasagot po please
answer it please​

Answers

Answer:

24 I hope help you yieeeeeee

Answer:

ummmmmmmmmmm itsssssssss

Step-by-step explanation:

What is 8 times larger than 3

Answers

8 x 3 = 24 for should be 24

Only because 8 times larger than three seems like a multiplication problem..

I hope this helps if it is correct!

Answer:

(8+1) x 3 = 27

Step-by-step explanation:

this is correct rather than 8 x 3 = 24 because both sides of 8x3=24 are equal rather than the sum being greater than the statement.

Jonnell has finished 20% of an art project that has taken him a total of 3 hours so far. If he continues to work at the same rate, how many hours will it take for him to complete the entire project?

Answers

Answer:

15 hours

Step-by-step explanation:

20%=3 hours

times 5 both sides

100%=15 hours

In each bouquet of flowers, there are 2 roses and 5 white carnations. Complete the table to find how many roses and carnations there are in 4 bouquets of flowers.

Answers

Answer:

28roses 70carnations

Step-by-step explanation:

4+6+8+10=28

10+15+20+25=70

Maybe i think

The greatest possible number whose digits are all even numbers from 1 to 9

Answers

Answer:

8642

Step-by-step explanation:

Our even numbers from 1-9 are:

2,4,6,8

The largest possible number using the even numbers once is 8642.

Hoped this helped

For which function is the ordered pair (5, 10) not a solution?

y = 15 - x
y = x - 5
y = x + 5
y = 2 x

Answers

Answer:

y = x - 5

Step-by-step explanation:

y = x - 5

10 = 5 - 5

10 is not equal 0

Answer:

y = x - 5

Step-by-step explanation:

y = x - 5

10 = 5 - 5

10 is not equal 0

24. A triangle has side lengths of 6, 8, and 9. What type of triangle is it?
acute
equiangular
obtuse
right

Answers

Answer:


It a acute

Hope this help you :)

(27/8)^1/3×[243/32)^1/5÷(2/3)^2]
Simplify this question sir pleasehelpme​

Answers

Step-by-step explanation:

[tex] = {( \frac{27}{8} )}^{ \frac{1}{3} } \times ( \frac{243}{32} )^{ \frac{1}{5} } \div {( \frac{2}{3} )}^{2} [/tex]

[tex] = { ({ (\frac{3}{2} )}^{3}) }^{ \frac{1}{3} } \times {( {( \frac{3}{2}) }^{5} )}^{ \frac{1}{5} } \div {( \frac{2}{3} )}^{2} [/tex]

[tex] = {( \frac{3}{2} )}^{3 \times \frac{1}{3} } \times {( \frac{3}{2} )}^{5 \times \frac{1}{5} } \times {( \frac{3}{2} )}^{2} [/tex]

[tex] = \frac{3}{2} \times \frac{3}{2} \times {( \frac{3}{2} )}^{2} [/tex]

[tex] = {( \frac{3}{2} )}^{1 + 1 + 2} [/tex]

[tex] = {( \frac{3}{2} )}^{4} \: or \: \frac{81}{16} [/tex]

[tex]\large\underline{\sf{Solution-}}[/tex]

[tex]\sf{\longmapsto{\bigg( \dfrac{27}{8} \bigg)^{\frac{1}{3}} \times \Bigg[\bigg( \dfrac{243}{32} \bigg)^{\frac{1}{5}} \div \bigg(\dfrac{2}{3} \bigg)^{2}\Bigg]}} \\[/tex]

We can write as :

27 = 3 × 3 × 3 = 3³

8 = 2 × 2 × 2 = 2³

243 = 3 × 3 × 3 × 3 × 3 = 3⁵

32 = 2 × 2 × 2 ×2 × 2 = 2⁵

[tex]\sf{\longmapsto{\bigg( \dfrac{3 \times 3 \times 3}{2 \times 2 \times 2} \bigg)^{\frac{1}{3}} \times \Bigg[\bigg( \dfrac{3 \times 3 \times 3 \times 3 \times 3}{2 \times 2 \times 2 \times 2 \times 2} \bigg)^{\frac{1}{5}} \div \bigg(\dfrac{2}{3} \bigg)^{2}\Bigg]}} \\[/tex]

[tex]\sf{\longmapsto{\bigg( \dfrac{{(3)}^{3}}{{(2)}^{3}} \bigg)^{\frac{1}{3}} \times \Bigg[\bigg( \dfrac{({3}^{5})}{{(2)}^{5}} \bigg)^{\frac{1}{5}} \div \bigg(\dfrac{2}{3} \bigg)^{2}\Bigg]}} \\[/tex]

Now, we can write as :

(3³/2³) = (3/2)³

(3⁵/2⁵) = (3/2)⁵

[tex]\sf{\longmapsto{\left\{\bigg(\frac{3}{2} \bigg)^{3} \right\}^{\frac{1}{3}} \times \Bigg[\left\{\bigg(\frac{3}{2} \bigg)^{5} \right\}^{\frac{1}{5}} \div \bigg(\dfrac{2}{3} \bigg)^{2}\Bigg]}} \\[/tex]

Now using law of exponent :

[tex]{\sf{({a}^{m})^{n} = {a}^{mn}}}[/tex]

[tex]\sf{\longmapsto{\bigg( \frac{3}{2} \bigg)^{3 \times \frac{1}{3}} \times \Bigg[\bigg(\frac{3}{2} \bigg)^{5 \times \frac{1}{5}} \div \bigg(\dfrac{2}{3} \bigg)^{2}\Bigg]}} \\[/tex]

[tex] \sf{\longmapsto{\bigg( \frac{3}{2} \bigg)^{\frac{3}{3}} \times \Bigg[\bigg(\frac{3}{2} \bigg)^{\frac{5}{5}} \div \bigg(\dfrac{2}{3} \bigg)^{2}\Bigg]}} \\[/tex]

[tex]\sf{\longmapsto{\bigg( \frac{3}{2} \bigg)^{1} \times\Bigg[\bigg(\frac{3}{2} \bigg)^{1} \div \bigg(\dfrac{2}{3} \bigg)^{2}\Bigg]}} \\[/tex]

[tex]\sf{\longmapsto{\bigg( \frac{3}{2} \bigg)^{1} \times \Bigg[\bigg(\frac{3}{2} \bigg)^{1} \times \bigg(\dfrac{3}{2} \bigg)^{2}\Bigg]}} \\[/tex]

[tex]\sf{\longmapsto{\bigg( \frac{3}{2} \bigg)^{1} \times \Bigg[\bigg(\frac{3}{2} \bigg)^{1} \times \bigg(\dfrac{3}{2} \times \dfrac{3}{2} \bigg)\Bigg]}} \\[/tex]

[tex]\sf{\longmapsto{\bigg( \dfrac{3}{2} \bigg)^{1} \times \Bigg[\bigg(\dfrac{3}{2} \bigg)^{1} \times \bigg(\dfrac{3 \times 3}{2 \times 2}\bigg)\Bigg]}} \\[/tex]

[tex]\sf{\longmapsto{\bigg( \dfrac{3}{2} \bigg)^{1} \times \Bigg[\bigg(\dfrac{3}{2} \bigg)^{1} \times \bigg(\dfrac{9}{4}\bigg)\Bigg]}} \\[/tex]

[tex] \sf{\longmapsto{\bigg( \frac{3}{2} \bigg)\times \Bigg[\bigg(\frac{3}{2} \bigg)\times \bigg(\dfrac{9}{4}\bigg)\Bigg]}} \\[/tex]

[tex]\sf{\longmapsto{\bigg( \dfrac{3}{2} \bigg)\times \Bigg[ \: \: \dfrac{3}{2} \times \dfrac{9}{4} \: \: \Bigg]}}\\[/tex]

[tex]\sf{\longmapsto{\bigg( \dfrac{3}{2} \bigg)\times \Bigg[ \: \: \dfrac{3 \times 9}{2 \times 4} \: \: \Bigg]}} \\[/tex]

[tex]\sf{\longmapsto{\bigg(\dfrac{3}{2} \bigg)\times \Bigg[ \: \: \dfrac{27}{8} \: \: \Bigg]}} \\[/tex]

[tex]\sf{\longmapsto{\dfrac{3}{2} \times \dfrac{27}{8}}} \\[/tex]

[tex]\sf{\longmapsto{\dfrac{3 \times 27}{2 \times 8}}} \\[/tex]

[tex] \sf{\longmapsto{\dfrac{81}{16}}\: ≈ \:5.0625\:\red{Ans.}} \\[/tex]

(2x+y)2-y2 if x=-3 y=4 and z=-5

Answers

Answer:
-12
Hope this helps!
So your equation it (2x+y)2-y2 so then you will substitute meaning your equation will now be 2(-3)+4*2-(4)2 and I assume it’s squared and there is a lot of things missing but I get -6+8-16 all together it would be 14 that’s what I get at least

Express the tan G as a fraction in simplest terms.

Answers

Answer:

[tex]\frac{\sqrt{70} }{5}[/tex]

help.....................

Answers

Answer:

Answers in Explanation

Step-by-step explanation:

First Question:

[tex]\sqrt{100}[/tex] + [18 ÷ 3 x 4 - 15] - (60 - 7^2 - 1)

[tex]\sqrt{100}[/tex] + [24 - 15]  - (60 - 49 - 1)

[tex]\sqrt{100}[/tex] + 9 - 10

10 + 9 - 10

Answer = 9

Second Question:

5x + 2x = 7x

5x^2 + 3x^2 = 8x^2

2x + 3x - x = 4x

2x + 3y + x + y = 3x + 4y

9x - 6x = 3x

-7y + 3x + 4x + 3y = 7x - 4y

-7x^2 + 2x^2 + 9x^2 = 8x^2

(3x^2 + 5x + 4) - (-1 + x^2) = 2x^2 + 5x + 5

(3 + 2x - x^2) + (x^2 + 8x + 5) = 10x + 8

(3x - 4) - (5x + 2) = -2x - 6

(2x^2 + 5x + 3) - (x^2 - 2x + 3) = x^2 + 7x

(3x^2 + 2x - 5) - (2x^2 - x - 4) = x^2 + 3x - 1

Third Question:

17x + 2y

(5x + 12y) + (3x + y) = 8x + 13y

17x - 8x = 9x

2y - 13y = -11y

Answer: 9x - 11y

Help me this question is so hard i fried up my brain yesterday working on it for so long!!!!

Answers

Hello there! (:

The answer is 9.

3^4=3*3*3*3 (81)

3^2=9

81:9=9

So the answer is 9.

Hope it helps! If you have any question or query, feel free to ask! (:

~An excited gal

[tex]SparklingFlower[/tex]

solve pls brainliest

Answers

Answer:

[tex]18 {m}^{2} [/tex]

Step-by-step explanation:

[tex]area \: = 6m \times 4m \\ = 24 {m}^{2} \\ \\ grass \: area = 3m \times 2m \\ = 6 {m}^{2} \\ \\ cement \: area \: = 24 {m}^{2} - 6 {m}^{2} \\ = 18 {m}^{2} [/tex]

Answer:

18 m^2

Step by step explanation:

In these types of math problems, we have two ways to solve.

1) Directly find the area of the shaded area.

2)Find the unshaded area and then minus that from the total area.

In this case, I will use the second way.

The grass area (unshaded) is 3 x 2 = 6 m^2 ( 6 square meters )

The total area (grass + cement) is 4 x 6 = 24 m^2 ( 24 square meters )

Now, we want the area of the cement part but the grass's area is also in the total.

So, we minus 6m^2 from 24m^2.

Then we get 18m^2.

And that is the answer.

I hope it helps.

(Note : because this problem is easy, you can use both ways but most use the second way. There may also be problems where we can use only the first or second way.)

Which of the following is true?

|−5| < 4
|−4| < |−5|
|−5| < |4|
|−4| < −5

Answers

Answer:

|-4| < |-5|

Step-by-step explanation:

because if modules is given sub sign will be deducate

3. A gym charges a fee of $15 per month plus an additional charge for every group class
attended. The total monthly gym cost T can be represented by this equation: T = 15+c*n,
where c is the additional charge for a group class, and n is the number of group classes
attended
Which equation can be used to find the number of group classes a customer attended if we
know c and T?
a. n = I - 15
N
b. n=1 – 150
c. n = (T - 15) - C.
(T-15)
d. n=
1

Answers

Answer:

Option D)  [tex]\huge\sf{n\:=\:\frac{(T\:-\:15)}{c}}[/tex]

Step-by-step explanation:

Given the equation, T = 15 + c × n, where:

T = represents the total monthly gym cost

c =  represents the additional charge for a group class, and

n = represents the number of group classes attended

Solution:

In order to determine which equation can be used to find the number of group classes a customer attended, if there are given values for c and T, we must isolate the variable, n  algebraically.

The first step is to subtract 15 from both sides:

T = 15 + c × n

T - 15 = 15 - 15 + c × n

T - 15 = c × n

Next, divide both sides by c to isolate n :

[tex]\huge\mathsf{\frac{({T\:-\:15})}{c}\:=\:\frac{{c\:\times\:n}}{c}}[/tex]

[tex]\huge\sf{n\:=\:\frac{(T\:-\:15)}{c}}[/tex]

Therefore, the correct answer is Option D) [tex]\huge\sf{n\:=\:\frac{(T\:-\:15)}{c}}[/tex].

Jonas has bought three t-shirts. Each t-shirt was the same price. After using a 10% off coupon the total charge was 95 what was the cost of each t-shirt

Answers

Answer:

35 5/27

Step-by-step explanation:

You just have to set up an equation:

0.9/95 = 1/x

And if you cross-multiply, you get:

0.9x=95

Which means x is 95/0.9 which is 105 5/9. So, if all three t-shirts costed 105 5/9, and they all costed the same, then one t-shirt costs 35 5/27.

the sum of three whole numbers in a row is 57. what are the three numbers?

Answers

Answer:

18, 19, 20

Step-by-step explanation:

(n) + (n + 1) + (n + 2) = 57

3n + 3 = 57

3n = 57 - 3 = 54

n = 54/3 = 18

three numbers are 18, 18+1, 18+2

18, 19, 20

Answer:

18,19,20

Step-by-step explanation:

x + (x+1) + (x+2) = 57

x + x + 1 + x + 2 = 57

Combine like terms

3x + 3 = 57

subtract 3 from both sides

3x = 54

divide both sides by 3

x = 18

The answer is 18, 19, 20

Since we know the value of x, we can look at it like this:

x + (x+1) + (x+2) --- > 18 + (18+ 1) + (18+2) --> 18 + 19 + 20

Can someone help plz

Answers

Here’s ur help The answer is 571


Consider this function.
h(x) = (x - 2)^2+3

Which of the following domain restrictions would enable h(x) to have an inverse function?
a. x < 1
b. x >5
c. x < 3
d. x > 4

(Ps: all four answer and larger equal then or smaller equal then

Answers

Answer:

No inverse function:  (a), (b), (c)

Inverse function exists:  (d)

Step-by-step explanation:

The graph of h(x) = (x - 2)^2 + 3 is a parabola that opens upward and has vertex at (2, 3).  If the entire graph is drawn, and the horizontal line test then applied, h(x) would not have an inverse, because the horizontal line would intersect the  parabolic graph twice.

Note that if we restricted the domain to x ≥ 2, the resulting graph would pass the horizontal line test.  This would also be true for x ≥ 3, x ≥ 4, and so on.  Not so for (a) x < 1.  False for x > -5.  True for x < 3.  True for x > 4.

No inverse function:  (a), (b), (c)

Inverse function exists:  (d)

what is the awnser and solve for C
2c=6–3c

Answers

[tex]▪▪▪▪▪▪▪▪▪▪▪▪▪  {\huge\mathfrak{Answer}}▪▪▪▪▪▪▪▪▪▪▪▪▪▪[/tex]

[tex]\fbox \colorbox{black}{ \colorbox{white}{C} \:  \:  \:   \:  \:  \:  \: \: \colorbox{white}{  = }  \:  \:  \:  \:  \:   + \colorbox{white}{6/5}}[/tex]

[tex] \large \boxed{ \mathfrak{Step\:\: By\:\:Step\:\:Explanation}}[/tex]

Let's find the value of c ~

[tex]2c = 6 - 3c[/tex]

[tex]2c + 3c = 6[/tex]

[tex]5c = 6[/tex]

[tex]c = \dfrac{6}{5} \: \: or \: \: 1.2[/tex]

Answer:

c= -6

Step-by-step explanation:

2c -6 = 3c

we move all to the left

2c -6 (3c)= 0

add everything and the variables

-1c - 6 = 0

move all terms containing c to the left everything else to the right

-1c = 6

c=6/-1

c=-6

which rotation about its center will carry a regular hexagon onto itself

Answers

There are 6 angles between neighbour vertices, they all are equal (because a hexagon is regular) and their sum is 360°. Thus each angle has a measure of 360°/6=60°. Each subsequent rotation by 60° also maps a hexagon onto itself.

Using the appropriate Algebraic identity evaluate the following:(4a - 5b)²​

Answers

Solution:

[tex](4a - 5b)^{2} \\ by \: \: \: using \: \: \: (x - y)^{2} = {x}^{2} - 2xy + {y}^{2} \\ = {(4a)}^{2} - 2(4a)(5b) + {(5b)}^{2} \\ = {16a}^{2} - 40ab + 25 {b}^{2} [/tex]

Answer:

[tex] {16a}^{2} - 40ab + {25b}^{2} [/tex]

Hope it helps.

Do comment if you have any query.

When there’s a power to the second outside of the parentheses, you multiply the entire equation by its self, so (4a - 5b)(4a-5b)=16a^2-40ab+25b^2

21. A square park has an area of 120 m2

a) What are the dimensions of the park? Give your answer to the nearest metre.
b) If fencing costs $18.50/m, how much would it cost to install a fence around the park?
Show your work

plsss help me quick :((

Answers

Answer:

120m2

$6.25

$4.50

85

57

12

12

32

54

69

87

89

12

34

58

HELP ME OUT PLEASE!!!
WILL GIVE BRAINLIEST!!!

Answers

Answer:

-4 9/10, -4 3/4, -4.2

Step-by-step explanation:

A. -4.9

B. -4.75

C. -4.2

ANSWER:

-4 9

10

-4 3

10

-4.2

Step-by-step explanation:

HOPE IT HELPSSSS

What is the common ratio of the sequence 3, 21, 147, … ?
7
Which formula can be used to find the nth term of the sequence 3, 21, 147, … ?


Use the given formula to find the indicated terms of the sequence.

a4 =
1029
a5 =
7203

Answers

It is 7, C,
A= 1029
B=7203

Answer:

What is the common ratio of the sequence 3, 21, 147, …?  

7

Which formula can be used to find the nth term of the sequence 3, 21, 147, …?

c)

Use the given formula to find the indicated terms of the sequence.

a4 = 1029

a5 = 7203

Step-by-step explanation:

Answers for all three questions<3

Can someone help me with this please​

Answers

Answer:

y=1/2x

Step-by-step explanation:

Answer:

10

9

8

7

6

5

4

3

2

1

0 1 2 3 4 5 6 7 8 9 10

Step-by-step explanation:

So the arrow is pointing at 10 and  5

Answer 10 5

Not 5

its 10 5

So the answer is 10 5

BRAINLIEST, 5 STARS, AND A THANKS FOR WHOEVER HELPS!

Which statement about the answer to this problem is most accurate?
​5\6−3\8=19\24​


The answer 19\24 is reasonable because both fractions are closer to 1\2 than they are to 1, making the difference close to 0.

The answer 19\24 is reasonable because ​5\6​ and 3\8​ are both closer to 1 than to 1\2, making the difference close to 0.

The answer 19\24 is not reasonable because ​5\6​ is closer to 1 than to ​1\2​, and ​3\8​ is close to ​1\2​, making the difference close to 1\2. ​

The answer​ 19\24 ​is not reasonable because ​5\6​ is closer to ​1\2​ than to 1, and ​3\8​ is closer to 0 than to ​1\2, making the difference close to​ ​1\2​.

Answers

Answer:

The answer 19\24 is not reasonable because ​5\6​ is closer to 1 than to ​1\2​, and ​3\8​ is close to ​1\2​, making the difference close to 1\2. ​

Step-by-step explanation:

Answer:

1/2 1 0

Step-by-step explanation:

6p - 5 =13
3
-3
12
15

Answers

Answer:

6p=13+5

6p=18

p=18/6

p=3

Can someone help’ it’s due today pls help pls pls

Answers

Answer:

MQP = 138

Step-by-step explanation:

Since the angle bisector always divides the angle in half exactly, we know both of the angle measures we have are the same, and set it up like this: 5x + 19 = 2x + 49 (subtract 2x from both sides) 3x + 19 = 49 (subtract 19 on both sides) 3x = 30 (divide by 3) x = 10

Substitute that x value (10) into the original equations in place of x for each angle to get MQN = 69 and NQP = 69

Add the two values to get 138. MQP = 138

Answer:

138

Step-by-step explanation:

QN bisects ∠MQP. ⇒ ∠MQN = ∠NQP

5x + 19 = 2x + 49

Subtract 19 from both sides

5x = 2x + 49 - 19

5x = 2x + 30

Subtract 2x from both sides

5x -2x = 30

3x = 30

x = 30/3

x = 10

∠MQN = 5x + 19

            = 5*10 + 19

            = 50 +19

             = 69

∠MQP = ∠MQN +∠NQP = 69 + 69 = 138

Other Questions
One important result of the French Revolution was thatA. France enjoyed a lengthy period of peace and prosperity. B. The church was restored to its former role and power in French government.C. Political power shifted to the bourgeoisie.D. France lost its spirit of nationalism 15. Describe and correct the error in finding the intercepts of the graph of the equation.6x + 9y= 186x + 9(0) = 186x = 186x + Oy 186(0) + 9y = 189y == 18X = 3y = 2The x-intercept is at (0,3), and the y-intercept is at (2, 0). Match each of the leg muscles with the correct label. 2 Which of the following describes the transformation from Figure 1 to Figure 2? On a coordinate plane, figure A B C D E has points (negative 3, 5), (negative 2, 5), (negative 1, 4), (negative 2, 3), (negative 5, 3). Figure A prime B prime C prime D prime E prime has points (2, 2), (3, 2), (4, 1), (3, 0), (0, 0). CLEAR CHECK translation 2 units to the right and 3 units down translation 3 units to the left and 2 units up translation 5 units to the right and 3 units down translation 5 units to the left and 3 units up Please help me thank you Fr+Cl=FeCl3 Colette launches an air rocket in the upward, positive direction. It launches with an initial velocity of 25.5 m/s. It accelerates in the downward, negative direction at a rate of 9.81 m/s2. After 3.5 seconds, what is the magnitude of the rocket's displacement? Please help asap! I will give brainliest for correct answer! graph using the slop and y intercept How far is it around the perimeter of crater lake? You notice that one of your coworkers is disposing of contaminated materials in the regular trash. You have seen her throw away her gloves, blood-soaked gauze, and tubes of blood in the trash receptacle that is used for paper products and other regular waste. What effect could your coworker's actions have on other employees and explain how the employee should have disposed of the contaminated materials mentioned above: How do you redeem a winning ticket lottery?A) scan on posB) validate on terminalC) picklist on POSD): redemption barcode entered in the pis if no touch screen cm nhn ca anh ch v on th trong on trch "t nc" . t nhn xt v trch nghim ca tui tr vi t nc "Nhng em bit khng....................................................... t nc ca nhn dn, t nc ca ca dao thn thoi" 4-What was the Wade- Davis Bill? Why did Lincoln veto it? Which scientist is credited with having the greatest contribution to early microscopy and was the first to observe and describe single-celled organisms? How often is USPS mail typically delivered to a business? I need to turn it in by 5pm Please Help!!!!! At the movies, you order 3 boxes of popcorn and a bottle of water. The cost of the bottle of water is $1.75. Your total cost is $9.25. Write an EQUATION to find the cost of one box of popcorn. find the 125th term of the number pattern 2,4,6,8...... Recombination (crossing over) during prophase I of meiosis occurs when: Select all of the following that are ordered pairs of the given function.(x) = 3 - 2x Simplify:-0.2sqrt(x^2)